Bạn chưa đăng nhập. Vui lòng đăng nhập để hỏi bài

Những câu hỏi liên quan
Trần Hữu Ngọc Minh
Xem chi tiết
Bexiu
22 tháng 8 2017 lúc 12:03

(x−y)2=(x+y)2−4xy=2012−4xy" role="presentation" style="border:0px; direction:ltr; display:inline-block; float:none; font-size:16.38px; line-height:0; margin:0px; max-height:none; max-width:none; min-height:0px; min-width:0px; padding:1px 0px; position:relative; white-space:nowrap; word-spacing:normal; word-wrap:normal" class="MathJax_CHTML mjx-chtml">

xy" role="presentation" style="border:0px; direction:ltr; display:inline-block; float:none; font-size:16.38px; line-height:0; margin:0px; max-height:none; max-width:none; min-height:0px; min-width:0px; padding:1px 0px; position:relative; white-space:nowrap; word-spacing:normal; word-wrap:normal" class="MathJax_CHTML mjx-chtml">, tương đương với việc ta tìm GTLN,GTNN của  hay cần tìm GTLN,GTNN của 

x≥y" role="presentation" style="border:0px; direction:ltr; display:inline-block; float:none; font-size:16.38px; line-height:0; margin:0px; max-height:none; max-width:none; min-height:0px; min-width:0px; padding:1px 0px; position:relative; white-space:nowrap; word-spacing:normal; word-wrap:normal" class="MathJax_CHTML mjx-chtml"> thì: 

|x−y|=x−y=x+y−2y=201−2y" role="presentation" style="border:0px; direction:ltr; display:inline-block; float:none; font-size:16.38px; line-height:0; margin:0px; max-height:none; max-width:none; min-height:0px; min-width:0px; padding:1px 0px; position:relative; white-space:nowrap; word-spacing:normal; word-wrap:normal" class="MathJax_CHTML mjx-chtml">

1≤y≤100" role="presentation" style="border:0px; direction:ltr; display:inline-block; float:none; font-size:16.38px; line-height:0; margin:0px; max-height:none; max-width:none; min-height:0px; min-width:0px; padding:1px 0px; position:relative; white-space:nowrap; word-spacing:normal; word-wrap:normal" class="MathJax_CHTML mjx-chtml"> nên: 

Lập luận đi ngược lại thì tìm được các cực trị

Nguyễn Thiều Công Thành
22 tháng 8 2017 lúc 15:10

dùng cô si thôi

\(a^4+b^2\ge2a^2b;b^4+c^2\ge2b^2c;c^4+a^2\ge2c^2a\)

\(a^2b^2+a^2\ge2a^2b;b^2c^2+b^2\ge2b^2c;c^2a^2+c^2\ge2c^2a\)

từ 2 cái trên =>\(\left(a^2+b^2+c^2\right)^2+3\left(a^2+b^2+c^2\right)\ge6\left(a^2b+b^2c+c^2a\right)\)

\(\Rightarrow\left(a^2+b^2+c^2\right)^2\ge3\left(a^2b+b^2c+c^2a\right)\)

\(\Rightarrow P\ge a^2+b^2+c^2+\frac{3\left(ab+bc+ca\right)}{\left(a^2+b^2+c^2\right)^2}\)

đặt t=a2+b2+c2\(\ge\frac{\left(a+b+c\right)^2}{3}=3\)

\(\Rightarrow\left[2\left(t-\frac{1}{2}\right)^2-\frac{19}{2}\right]\left(t-3\right)\ge0\)

\(\Leftrightarrow2t^3-8t^2-3t+27\ge0\)

\(\Leftrightarrow\frac{2t^3-3t+27}{2t^2}\ge4\Rightarrow P\ge4\)

Aurora
Xem chi tiết
Nguyễn Việt Lâm
12 tháng 5 2021 lúc 22:15

\(\sqrt{2a^2+ab+2b^2}=\sqrt{\dfrac{3}{2}\left(a^2+b^2\right)+\dfrac{1}{2}\left(a+b\right)^2}\ge\sqrt{\dfrac{3}{4}\left(a+b\right)^2+\dfrac{1}{2}\left(a+b\right)^2}=\dfrac{\sqrt{5}}{2}\left(a+b\right)\)

Tương tự:

\(\sqrt{2b^2+bc+2c^2}\ge\dfrac{\sqrt{5}}{2}\left(b+c\right)\) ; \(\sqrt{2c^2+ca+2a^2}\ge\dfrac{\sqrt{5}}{2}\left(c+a\right)\)

Cộng vế với vế:

\(P\ge\sqrt{5}\left(a+b+c\right)\ge\dfrac{\sqrt{5}}{3}\left(\sqrt{a}+\sqrt{b}+\sqrt{c}\right)^3=\dfrac{\sqrt{5}}{3}\)

Dấu "=" xảy ra khi \(a=b=c=\dfrac{1}{9}\)

Lê An Bình
Xem chi tiết
Nguyễn Thị Quỳnh Như
18 tháng 4 2016 lúc 14:53

 Ta có : \(\sqrt{\frac{ab}{ab+2c}}=\sqrt{\frac{ab}{ab+\left(a+b+c\right)c}}=\sqrt{\frac{ab}{\left(a+c\right)\left(b+c\right)}}\le\frac{1}{2}\left(\frac{a}{a+c}+\frac{b}{b+c}\right)\)

Đẳng thức xảy ra khi và chỉ khi \(\frac{a}{a+c}+\frac{b}{b+c}\)

Tương tự ta cũng có 

           \(\sqrt{\frac{bc}{bc+2a}}\le\frac{1}{2}\left(\frac{b}{b+a}+\frac{c}{c+a}\right);\sqrt{\frac{ca}{ca+2b}}\le\frac{1}{2}\left(\frac{c}{c+a}+\frac{a}{a+b}\right)\)

Cộng các vế ta được \(S\le\frac{1}{2}\left(\frac{a+b}{a+b}+\frac{b+c}{b+c}+\frac{c+a}{c+a}\right)=\frac{3}{2}\)

Đẳng thức xảy ra khi và chỉ khi \(a=b=c=\frac{2}{3}\)

Vậy \(S_{max}=\frac{3}{2}\Leftrightarrow x=y=z=\frac{2}{3}\)

Cố gắng hơn nữa
Xem chi tiết
Thắng Nguyễn
19 tháng 5 2018 lúc 13:53

sos là ra ez

Cố gắng hơn nữa
19 tháng 5 2018 lúc 14:11

là sao ?

Thắng Nguyễn
19 tháng 5 2018 lúc 18:43

a)\(a^2+b^2+c^2\ge ab^2+bc^2+ca^2\)

\(\Leftrightarrow\left(a^2+b^2+c^2\right)\left(a+b+c\right)\ge3\left(ab^2+bc^2+ca^2\right)\)

\(\Leftrightarrow a^3+b^3+c^3+a^2b+b^2c+c^2a-2\left(ab^2+a^2c+bc^2\right)\ge0\)

\(\Leftrightarrow\left(c^2a-2ca^2+a^3\right)+\left(a^2b-2ab^2+b^3\right)+\left(b^2c-2bc^2+c^3\right)\ge0\)

\(\Leftrightarrow a\left(c^2-2ca+a^2\right)+b\left(a^2-2ab+b^2\right)+c\left(b^2-2bc+c^2\right)\ge0\)

\(\Leftrightarrow a\left(c-a\right)^2+b\left(a-b\right)^2+c\left(b-c\right)^2\ge0\) (luôn đúng)

Dấu "=" <=>a=b=c=1

Câu b để sau đi trời nóng mà máy gõ mãi ko xong 1 dòng chán quá

Thanh Tu Nguyen
Xem chi tiết
Nguyễn Ngọc Anh Minh
9 tháng 11 2023 lúc 8:16

 

1/\(=4a^2+4b^2+c^2+8ab-4bc-4ca+4b^2+4c^2+a^2+8bc-4ca-4ab+4a^2+4c^2+b^2+8ca-4bc-4ab=\)

\(=9a^2+9b^2+9c^2=9\left(a^2+b^2+c^2\right)\)

2/

Ta có

\(\left(a+b+c\right)^2=a^2+b^2+c^2+2\left(ab+bc+ca\right)\ge0\)

\(\Leftrightarrow a^2+b^2+c^2\ge-2\left(ab+bc+ca\right)=2\)

\(\Rightarrow P=9\left(a^2+b^2+c^2\right)\ge18\)

\(\Rightarrow P_{min}=18\)

Nguyễn  Thanh Hải
Xem chi tiết
Nguyễn Mai
Xem chi tiết
Liêu Nguyễn Công
Xem chi tiết
Thanh Ngô Thi
27 tháng 5 2016 lúc 10:31

Sử dụng giả thiết ax−by=√3ax−by=3 ta có:

(a2+b2)(x2+y2)=(ax+by)2+(ax−by)2=(ax+by)2+3(a2+b2)(x2+y2)=(ax+by)2+(ax−by)2=(ax+by)2+3

Áp dụng bất đẳng thức CauchyCauchy , suy ra:

a2+b2=x2+y2=(a2+b2)+(x2+y2)≥2√(a2+b2)(x2+y2)=2√(ax+by)2+3a2+b2=x2+y2=(a2+b2)+(x2+y2)≥2(a2+b2)(x2+y2)=2(ax+by)2+3

Do đó, ta đưa về bài toán tìm GTNN của: 2√x2+3+x2x2+3+x trong đó x=ax+byx=ax+by

Ta có:

(2√x2+3+x)2=4(x2+3)+4x√x2+3+x2=(x2+3)+4x√x2+3+4x2+9=(√x2+3+2x)2+9≥9(2x2+3+x)2=4(x2+3)+4xx2+3+x2=(x2+3)+4xx2+3+4x2+9=(x2+3+2x)2+9≥9

⇒2√x2+3+x≥3⇒2x2+3+x≥3

Vậy MinT=3MinT=3

Phan Cả Phát
27 tháng 5 2016 lúc 10:34

Sử dụng giả thiết ax−by=√3ax−by=3 ta có:

(a2+b2)(x2+y2)=(ax+by)2+(ax−by)2=(ax+by)2+3(a2+b2)(x2+y2)=(ax+by)2+(ax−by)2=(ax+by)2+3

Áp dụng bất đẳng thức CauchyCauchy , suy ra:

a2+b2=x2+y2=(a2+b2)+(x2+y2)≥2√(a2+b2)(x2+y2)=2√(ax+by)2+3a2+b2=x2+y2=(a2+b2)+(x2+y2)≥2(a2+b2)(x2+y2)=2(ax+by)2+3

Do đó, ta đưa về bài toán tìm GTNN của: 2√x2+3+x2x2+3+x trong đó x=ax+byx=ax+by

Ta có:

(2√x2+3+x)2=4(x2+3)+4x√x2+3+x2=(x2+3)+4x√x2+3+4x2+9=(√x2+3+2x)2+9≥9(2x2+3+x)2=4(x2+3)+4xx2+3+x2=(x2+3)+4xx2+3+4x2+9=(x2+3+2x)2+9≥9

⇒2√x2+3+x≥3⇒2x2+3+x≥3

Vậy MinT=3MinT=3

Võ Đông Anh Tuấn
27 tháng 5 2016 lúc 11:23

Không mất tính tổng quát, giả sử b là số nằm giữa a và c, khi đó ta có c(a − b)(b − c) ≥ 0, tương đương

         \(a^2b+b^2c+c^2a\le b\left(a^2+ca+c^2\right)\)

Từ đó, kết hợp với bất đẳng thức AM − GM bộ ba số, ta có :

        \(\left(a^2b+b^2c+c^2a\right)\left(ab+bc+ca\right)\le b\left(a^2+ca+c^2\right)\left(ab+bc+ca\right)\)

                                                                               \(\le\frac{\left(3b+a^2+ca+c^2+ab+bc+ca\right)^3}{3^4}\)

                                                                                \(=\frac{\left(\left(a+c\right)^2+3b+ab+bc\right)^2}{3^4}\)

                                                                                  \(=\frac{\left(\left(3-b\right)^2+3b+b\left(3-b\right)\right)^3}{3^4}\)

                                                                                    \(=9\)

Mặt khác, theo bất đẳng thức Cauchy − Schwarz thì :

                  \(a^4+b^4+c^4\ge\frac{\left(a^2+b^2+c^2\right)^2}{3}\)

Do đó, sử dụng các đánh giá trên, sau đó liên tục dùng Cauchy − Schwarz ta có :

  \(P\ge\frac{7}{3}\left(a^2+b^2+c^2\right)^2+\frac{\left(ab+bc+ca\right)^2}{9}\)

  \(=\frac{41}{18}\left(a^2+b^2+c^2\right)^2+\frac{\left(a^2+b^2+c^2\right)^2+\left(ab+bc+ca\right)^2+\left(ab+bc+ca\right)^2}{18\times3}\)

\(\ge\frac{41}{18}\times\frac{\left(a+b+c\right)^4}{3^2}+\frac{\left(a^2+b^2+c^2+ab+bc+ca+ab+bc+ca\right)^2}{18\times3}\)

\(=\frac{22}{81}\left(a+b+c\right)^4\)

\(=22\)

Đẳng thức xảy ra khi và chỉ khi a = b = c = 1 nên giá trị nhỏ nhất của P là 22. 

tnt
Xem chi tiết
Yen Nhi
8 tháng 2 2023 lúc 13:18

Theo đề ra, ta có:

\(a^2+b^2+c^2\)

\(=\left(a+b+c\right)\left(a^2+b^2+c^2\right)\)

\(=a^3+b^3+c^3+a^2b+b^2c+c^2a+ab^2+bc^2+ca^2\)

Theo BĐT Cô-si:

\(\left\{{}\begin{matrix}a^3+ab^2\ge2a^2b\\b^3+bc^2\ge2b^2c\\c^3+ca^2\ge2c^2a\end{matrix}\right.\Rightarrow a^2+b^2+c^2\ge3\left(a^2b+b^2c+c^2a\right)\)

Do vậy \(M\ge14\left(a^2+b^2+c^2\right)+\dfrac{3\left(ab+bc+ac\right)}{a^2+b^2+c^2}\)

Ta đặt \(a^2+b^2+c^2=k\)

Luôn có \(3\left(a^2+b^2+c^2\right)\ge\left(a+b+c\right)^2=1\)

Vì thế nên \(k\ge\dfrac{1}{3}\)

Khi đấy:

\(M\ge14k+\dfrac{3\left(1-k\right)}{2k}=\dfrac{k}{2}+\dfrac{27k}{2}+\dfrac{3}{2k}-\dfrac{3}{2}\ge\dfrac{1}{3}.\dfrac{1}{2}+2\sqrt{\dfrac{27k}{2}.\dfrac{3}{2k}}-\dfrac{3}{2}=\dfrac{23}{3}\)

\(\Rightarrow Min_M=\dfrac{23}{3}\Leftrightarrow a=b=c=\dfrac{1}{3}\).